aras2213

New Member
ارسال ها
216
لایک ها
228
امتیاز
0
پاسخ : ماراتن نظریه ی اعداد (سطح پیشرفته)


سوال بعد:
همه چند جمله ای های
را بیابید که برای هر
، وجود داشته باشد
که


برای هر عدد اول و دلخواه مثل q داریم:
اگر درجه f بیشتر از 1 باشه اون وقت مقادیری که مشتق اون به ازای اعداد طبیعی میگیره بینهایت عامل اول دارن.چون درجه مشتقش حداقل 1 میشه.مثلا فرض کنید :




این یعنی
و همچنین
به پیمانه q^2 همنهشت نیستند.حالا نشون میدیم که این شرط سوال رو نقض میکنه.
اگر
و b بین 1 تا q^2 تغییر کنه،نتیجه میشه که
به ازای
همه باقیمانده های ممکن رو به پیمانه q^2 میده.یعنی:



اما برای
چنین چیزی برقرار نیست.پس درجه f حداکثر 1 هه.این جا به بعدش هم زیاد سخت نیست.درسته؟

---- دو نوشته به هم متصل شده است ----

راه حلتون درسته و اگه اون نامساوي رو هم چك كنيد جواب
بدست مياد ولي اون نامساوي هايي كه نوشتين همشون بايد حالت تساوي هم داشته باشن چون اگه اينجوري نباشه يه جوابي كه سوال داره بدست نمياد.
سوال بعد:
اعدادي صحيح و مثبت هستند به طوريكه
ثابت كنيد
عددي مركب است.
http://artofproblemsolving.com/community/c5h1083472p4774020
 

Dadgarnia

New Member
ارسال ها
1,350
لایک ها
1,127
امتیاز
0
پاسخ : ماراتن نظریه ی اعداد (سطح پیشرفته)

برای هر عدد اول و دلخواه مثل q داریم:
اگر درجه f بیشتر از 1 باشه اون وقت مقادیری که مشتق اون به ازای اعداد طبیعی میگیره بینهایت عامل اول دارن.چون درجه مشتقش حداقل 1 میشه.مثلا فرض کنید :



این یعنی
و همچنین
به پیمانه q^2 همنهشت نیستند.حالا نشون میدیم که این شرط سوال رو نقض میکنه.
اگر
و b بین 1 تا q^2 تغییر کنه،نتیجه میشه که
به ازای
همه باقیمانده های ممکن رو به پیمانه q^2 میده.یعنی:


اما برای
چنین چیزی برقرار نیست.پس درجه f حداکثر 1 هه.این جا به بعدش هم زیاد سخت نیست.درسته؟

---- دو نوشته به هم متصل شده است ----


Community - Art of Problem Solving
به نظر من كه درسته. لطفا سوال بعد رو هم بذار.
 

aras2213

New Member
ارسال ها
216
لایک ها
228
امتیاز
0
پاسخ : ماراتن نظریه ی اعداد (سطح پیشرفته)

همه اعداد طبیعی a,b رو پیدا کنید که
هر دو مکعب کامل باشند.
 

Dadgarnia

New Member
ارسال ها
1,350
لایک ها
1,127
امتیاز
0
پاسخ : ماراتن نظریه ی اعداد (سطح پیشرفته)

همه اعداد طبیعی a,b رو پیدا کنید که
هر دو مکعب کامل باشند.
فرض مي كنيم
با جمع اين دو نامساوي بدست مياد:
كه تناقضه پس حداقل يكي از اين دو نامساوي غلطه. فرض مي كنيم
پس داريم:

از طرفي براي
داريم
پس كافيه حالات خاص رو بررسي كنيم كه ديگه كاري نداره.

---- دو نوشته به هم متصل شده است ----

سوال بعد:
همه اعداد اول
را بيابيد به طوريكه
.
 

TheOverlord

New Member
ارسال ها
159
لایک ها
282
امتیاز
0
پاسخ : ماراتن نظریه ی اعداد (سطح پیشرفته)

سوال بعد:
همه اعداد اول
را بيابيد به طوريكه
.
ابتدا فرض كنيد هر سه عدد فردند.
دقت كنيد كه اين اعداد بديهتا دو به دو نسبت به هم اول، و نتيجتا متمايزند.
حال واضح است كه

حال اگر هر سه عدد دو نباشد، بنابر تقارن
آنگاه اگر
آنگاه داريم

كه تناقض است. بنابرين
و به طريق مشابه
پس اگر بزرگترين عدد از بين اين ٣ بنا به تقارن
باشد چون
كه تناقض است. بنابرين هر سه مرتبه برابر دو هستند.
پس
اما اين اعداد فرد هستند، پس اگر بزرگترينشان
باشد دوباره بخاطر
به تناقض ميرسيم.
پس يكي از اين اعداد اول فرد نيست پس ٢ است. بنا به تقارن
حال بديهتا باقي اعداد فردند.دقت كنيد كه
حال اگر
بديهتا
پس در اين حالت مساله حل است. اما اگر

كه تناقض است. پس. تنها جوابها همان
هستند.
سوال بعد: فرض كنيد جمع مقسوم عليه هاي عدد
برابر
باشد. ثابت كنيد براي يك عدد طبيعي ثابت مثل
بينهايت عدد طبيعي
وجود دارند كه در خاصيت
صدق كنند.​
 
آخرین ویرایش توسط مدیر

Dadgarnia

New Member
ارسال ها
1,350
لایک ها
1,127
امتیاز
0
پاسخ : ماراتن نظریه ی اعداد (سطح پیشرفته)

ابتدا فرض كنيد هر سه عدد فردند.
دقت كنيد كه اين اعداد بديهتا دو به دو نسبت به هم اول، و نتيجتا متمايزند.
حال واضح است كه

حال اگر هر سه عدد دو نباشد، بنابر تقارن
آنگاه اگر
آنگاه داريم

كه تناقض است. بنابرين
و به طريق مشابه
پس اگر بزرگترين عدد از بين اين ٣ بنا به تقارن
باشد چون
كه تناقض است. بنابرين هر سه مرتبه برابر دو هستند.
پس
اما اين اعداد فرد هستند، پس اگر بزرگترينشان
باشد دوباره بخاطر
به تناقض ميرسيم.
پس يكي از اين اعداد اول فرد نيست پس ٢ است. بنا به تقارن
حال بديهتا باقي اعداد فردند.دقت كنيد كه
حال اگر
بديهتا
پس در اين حالت مساله حل است. اما اگر

كه تناقض است. پس. تنها جوابها همان
هستند.
سوال بعد: فرض كنيد جمع مقسوم عليه هاي عدد
برابر
باشد. ثابت كنيد براي يك عدد طبيعي ثابت مثل
بينهايت عدد طبيعي
وجود دارند كه در خاصيت
صدق كنند.​
وقتي كه
بديهتا
شما چه جوري نتيجه گرفتيد كه
؟ در ضمن هر دو حالت
و
بايد بررسي بشن و توي آخر راه حل هم يه اشتباه كوچيك كردين جواباش
و جايگشت هاي دوريش هستند.
 

TheOverlord

New Member
ارسال ها
159
لایک ها
282
امتیاز
0
پاسخ : ماراتن نظریه ی اعداد (سطح پیشرفته)

وقتي كه
بديهتا
شما چه جوري نتيجه گرفتيد كه
؟ در ضمن هر دو حالت
و
بايد بررسي بشن و توي آخر راه حل هم يه اشتباه كوچيك كردين جواباش
و جايگشت هاي دوريش هستند.
از اونجا كه مرتبه برابر دو هست، پس برابر يك نيست، پس
ويرايش كردم تا درست شه.
ثابت كردم از
به تناقض ميرسيم. به طريق مشابه از
هم به تناقض ميرسيم.
جواب آخر رو هم ويرايش كردم.​
 
آخرین ویرایش توسط مدیر

aras2213

New Member
ارسال ها
216
لایک ها
228
امتیاز
0
پاسخ : ماراتن نظریه ی اعداد (سطح پیشرفته)

سوال بعد: فرض كنيد جمع مقسوم عليه هاي عدد
برابر
باشد. ثابت كنيد براي يك عدد طبيعي ثابت مثل
بينهايت عدد طبيعي
وجود دارند كه در خاصيت
صدق كنند.​
فرض کنید
کوچکترین عدد اول بزرگتر از
باشه و
به ترتیب کوچکترین اعداد اول بزرگتر از
باشن به طوری که


که چون جمع معکوس اعداد اول نامتناهی است، t وجود دارد.طبق باقیمانده چینی r وجود داره که


حالا تصاعد
بینهایت عدد اول داره(طبق دیریشله)فرض کنید m یکی از این اعداد اول باشه که از a بزرگتر باشه.

در این صورت:
.
همچنین:
.پس:



پس هر یک از این m ها یه جواب برای سواله.
 
آخرین ویرایش توسط مدیر

nima-1376

New Member
ارسال ها
63
لایک ها
53
امتیاز
0
پاسخ : ماراتن نظریه ی اعداد (سطح پیشرفته)

نشان دهید بینهایت عدد طبیعی
هست که
خالی از مربع باشد.
 

nima-1376

New Member
ارسال ها
63
لایک ها
53
امتیاز
0
پاسخ : ماراتن نظریه ی اعداد (سطح پیشرفته)

سوال اول : نشان دهید بی نهایت
هست که


سوال دوم: فرض کنید
عددی طبیعی است. نشان دهید بیشمار عدد طبیعی
هست به طوری که


سوال سوم :100 عدد طبیعی متمایز داریم زیر هر کدام ب.م.م 99 تای دیگر را مینویسیم. حداکثر چند تا از این اعداد متمایزند؟

سوال چهارم: نشان دهید تعداد اعداد اولی که میتوان عدد دلخواه
را در بین ارقام آن به صورت متوالی یافت نامتناهی است!
 

Dadgarnia

New Member
ارسال ها
1,350
لایک ها
1,127
امتیاز
0
پاسخ : ماراتن نظریه ی اعداد (سطح پیشرفته)

سوال اول : نشان دهید بی نهایت
هست که


سوال دوم: فرض کنید
عددی طبیعی است. نشان دهید بیشمار عدد طبیعی
هست به طوری که


سوال سوم :100 عدد طبیعی متمایز داریم زیر هر کدام ب.م.م 99 تای دیگر را مینویسیم. حداکثر چند تا از این اعداد متمایزند؟

سوال چهارم: نشان دهید تعداد اعداد اولی که میتوان عدد دلخواه
را در بین ارقام آن به صورت متوالی یافت نامتناهی است!
١- قرار بدين
.
٢- اگه
زوج باشه
حداقل يه عامل اول و فرد مثل
داره حالا بنابر لم دو خط براي
و
دلخواه داريم:

اگه
فرد باشه
حداقل يه عامل اول و فرد مثل
داره حالا بنابر لم دو خط براي
و
دلخواه داريم:




---- دو نوشته به هم متصل شده است ----

٣-
اعدادي اول و متمايز هستند قرار بدين
در اينصورت زير
،
رو بايد بنويسيم و بديهتا همه ي اعداد با هم متمايزند. درسته؟
 

Dadgarnia

New Member
ارسال ها
1,350
لایک ها
1,127
امتیاز
0
پاسخ : ماراتن نظریه ی اعداد (سطح پیشرفته)

سوال چهارم: نشان دهید تعداد اعداد اولی که میتوان عدد دلخواه
را در بین ارقام آن به صورت متوالی یافت نامتناهی است!
لطفا يكي جواب اينو بذاره :|
 

MNikdan

New Member
ارسال ها
124
لایک ها
52
امتیاز
0
پاسخ : ماراتن نظریه ی اعداد (سطح پیشرفته)

لطفا يكي جواب اينو بذاره :|
طبق قضیه ی دیریکله میدونیم که اگه
اون وقت به ازای بی نهایت
عدد
اول خواهد بود.
فرض میکنیم
تعداد ارقام
باشه. حالا کافیه قرار بدیم
و
ُ . واضحه که
پس طبق دیریکله بی نهایت عدد اول به صورت
داریم که توی همه ی این اعداد عدد
به طور کامل دیده میشه...
 

Dadgarnia

New Member
ارسال ها
1,350
لایک ها
1,127
امتیاز
0
پاسخ : ماراتن نظریه ی اعداد (سطح پیشرفته)

طبق قضیه ی دیریکله میدونیم که اگه
اون وقت به ازای بی نهایت
عدد
اول خواهد بود.
فرض میکنیم
تعداد ارقام
باشه. حالا کافیه قرار بدیم
و
ُ . واضحه که
پس طبق دیریکله بی نهایت عدد اول به صورت
داریم که توی همه ی این اعداد عدد
به طور کامل دیده میشه...
احتمالا منظور ايشون راه حل مقدماتي بوده چون وبلاگ بچه هاي 26 هم يه بار اين سوالو گذاشته بودن و گفته بودن راه حل مقدماتي داره.

---- دو نوشته به هم متصل شده است ----

حالا شما يه سوال بذارين تا به ماراتن ادامه بديم.
 

MNikdan

New Member
ارسال ها
124
لایک ها
52
امتیاز
0
پاسخ : ماراتن نظریه ی اعداد (سطح پیشرفته)

آخه اسم تاپیک ماراتن پیشرفته بود. منم فکر کردم مشکلی نداره... هر چند که دیریکله خیلی هم پیشرفته نیست !!!
به هر حال :
1) ثابت کنید بی نهایت
وجود داره که تمام عوامل اول
از
کوچکتر باشن.

2)فرض کنید
برابر حاصل ضرب عوامل اول متمایز
باشه. دنباله
از اعداد طبیعی بزرگتر از 1 به صورت زیر تعریف میشه:

که
دلخواهه. ثابت کنید به ازای هر
دنباله ای حسابی به طول
از جمله های متوالی این دنباله وجود داره.
 

Dadgarnia

New Member
ارسال ها
1,350
لایک ها
1,127
امتیاز
0
پاسخ : ماراتن نظریه ی اعداد (سطح پیشرفته)

آخه اسم تاپیک ماراتن پیشرفته بود. منم فکر کردم مشکلی نداره... هر چند که دیریکله خیلی هم پیشرفته نیست !!!
به هر حال :
1) ثابت کنید بی نهایت
وجود داره که تمام عوامل اول
از
کوچکتر باشن.

2)فرض کنید
برابر حاصل ضرب عوامل اول متمایز
باشه. دنباله
از اعداد طبیعی بزرگتر از 1 به صورت زیر تعریف میشه:

که
دلخواهه. ثابت کنید به ازای هر
دنباله ای حسابی به طول
از جمله های متوالی این دنباله وجود داره.
من منظورم نظریه اعداد مقدماتی بود نه مقدماتی و پیشرفته ای که اینجا تعریف شده.
1) کافیه ثابت کنیم معادله
بی نهایت جواب طبیعی داره. این معادله ی پله و چون یه جواب نابدیهی مثل
داره بی نهایت جواب براش وجود داره.
 

Dadgarnia

New Member
ارسال ها
1,350
لایک ها
1,127
امتیاز
0
پاسخ : ماراتن نظریه ی اعداد (سطح پیشرفته)

سوال بعد:
ثابت کنید اعداد صحیح
که قدر مطلق همه بزرگتر از ۱۰۰۰۰۰۰ است وجود دارند كه
.
 
آخرین ویرایش توسط مدیر
ارسال ها
288
لایک ها
154
امتیاز
43
پاسخ : ماراتن نظریه ی اعداد (سطح پیشرفته)

سوال بعد:
ثابت کنید اعداد صحیح
که همه بزرگتر از ۱۰۰۰۰۰۰ هستند وجود دارند که جمع معکوسات
برابر با معکوس
باشد.


اينطوري ميشه! مطمينيد سوال درسته؟
 
بالا